В чем отличие проводников от диэлектриков: в чем отличие проводников от диэлектриков

Содержание

Чем отличаются диэлектрики от проводников?

В соответствии с электрическими свойствами существуют два противоположных типа веществ. Это проводники и диэлектрики. Рассмотрим, чем отличаются диэлектрики от проводников. Проводниками называют тела, которые проводят электрический заряд от заряженного тела к незаряженному (или имеющему меньший заряд). Свойство проводить заряд через себя объясняется тем, что проводники имеют свободные носители заряда. Так, в металлах такими свободными носителями заряда являются электроны. В противоположность проводникам, диэлектрики являются изоляторами. Изоляторы – это вещества сквозь которые заряды не могут переходить от одного тела к другому. Яркими примерами диэлектриков являются: сухое дерево, эбонит, стекло.
Если тело из проводящего вещества поместить в электрическое поле, то произойдет разделение зарядов противоположных знаков в теле. Это явление носит название — электростатическая индукция. При внесении проводника в электрическое поле его свободные заряды приходят в движение.

Они перераспределяются , что изменяет электрическое поле. Перемещение зарядов идет до тех пор, пока напряженность электрического поля внутри проводника не становится равной нулю. При этом свободные заряды распределяются на поверхности проводника так, что вектор напряженности электрического поля становится перпендикулярным его поверхности.  Поверхность проводника, не зависимо от формы, является эквипотенциальной в электрическом поле.
В диэлектрике, который помещен в электрическое поле, заряды не могут разделиться, там нет свободных зарядов. В электрическом поле происходит поляризация диэлектрика. Поляризация – это процесс смещения зарядов разных знаков в противоположные стороны (смещение происходит в пределах атома или молекулы). Результат поляризации проявляется в возникновении на поверхности диэлектрика связанных зарядов. При этом, вектор напряженности поля, которое создают связанные заряды, имеет направление противоположное вектору напряженности внешнего поля. При этом говорят, что диэлектрик ослабляет электрическое поле в раз по сравнению с тем же полем в вакууме.
— диэлектрическая проницаемость вещества.

Диэлектрики отличие от полупроводников — Энциклопедия по машиностроению XXL

Однако в отличие от полупроводников во многих диэлектриках подвижность электронов и дырок чрезвычайно мала в сотни и даже в тысячи раз ниже, чем в полупроводниках. Столь низкие значения подвижности связаны с тем, что электроны находятся в этих веществах в связанном состоянии, образуя квазичастицы— поляроны.  
[c.273]

Электропроводность диэлектриков в отличие от полупроводников чаще всего носит не электронный, а ионный характер. Это связано с тем, что ширина запрещенной зоны в диэлектриках АЯ >кГ и лишь ничтожное количество  [c.98]


Диэлектрики в отличие от полупроводников имеют более широкую запрещенную зону (до 7—10 эВ). Поэтому при обычных температурах они обладают очень низкой концентрацией свободных носителей заряда, обусловливающей чрезвычайно малую их электропроводность.
Это позволяет использовать диэлектрические пленки в качестве изолирующих прокладок между металлами или металлами и полупроводниками в тонкопленочных и интегральных схемах.  [c.271]

В отличие от металлов в полупроводниках и диэлектриках также возникает так называемый внутренний фотоэффект, состояш,ий в возбуждении электронов из валентной зоны в зону проводимости. Для внутреннего фотоэффекта энергия поглощенного светового кванта не должна быть меньше ширины запрещенной зоны (разность энергии между нижней границей зоны проводимости и верхней границей валентной зоны).  

[c.345]

В отличие от диэлектриков и полупроводников в металлах валентная зона заполнена электронами либо частично, либо целиком, но при этом перекрывается со следующей разрешенной зоной. Заполненные состояния от незаполненных отделяются уровнем Ферми. Таким образом, уровень Ферми в металлах расположен в разрешенной зоне.  [c.255]

См/м) н проводимостью диэлектриков (ss 10 + 10 ° См/м). Таким образом, естественные полупроводники отличаются от диэлектриков более узкой запрещенной зоной. У диэлектриков ширина запрещенной зоны составляет несколько электрон-вольт, а у полупроводников-около 1 эВ. Например, у кремния и германия ширина запрещенных зон равна соответственно 1,1 и 0,75 эВ.  

[c.342]

Приближенная количественная оценка показывает, что в диэлектрике с шириной запрещенной зоны 3 эВ концентрация свободных носителей заряда при комнатной температуре должна составлять j 2 10 м . При подвижности носителей Ыр 10 м /(В с) (100 см /(В с)) удельная электропроводность такого диэлектрика должна быть порядка 7 10 Ом х X м» (7-10 Oм см ). В действительности столь низкая электропроводность в диэлектриках не наблюдается из-за наличия в них примесей и дефектов, создающих энергетические уровни в запрещенной зоне. Концентрация свободных носителей заряда в таких диэлектриках определяется фактически количеством и характером расположения донорных и акцепторных уровней в запрещенной зоне.

У контакта же с металлом концентрация свободных носителей может существенно отличаться от концентрации в толще диэлектрика вследствие образования здесь слоев обогащения или обеднения. С подобным явлением мы уже встречались в гл. 8 при рассмотрении контакта металл — полупроводник.  [c.272]

Полупроводниками называют вещества, удельное сопротивление которых при нормальной температуре находится между значениями удельных сопротивлений проводников и диэлектриков (в диапазоне от до 10 °…10 Омом). Основным свойством полупроводника является зависимость его электропроводности от воздействия температуры, электрического поля, излучения и других факторов. Полупроводники в отличие от проводников имеют отрицательный коэффициент температурного удельного сопротивления, электропроводность полупроводников с увеличением температуры растет экспоненциально.  

[c.334]


Диэлектрики и полупроводники качественно подобны и те и другие имеют энергетическую щель в спектре электронных состояний.
Однако в полупроводниках эта щель (запрещенная зона) гораздо меньше. Поэтому проводимость полупроводников заключена в широком интервале, разделяющем проводимость металлов и диэлектриков. Например, для кремния при 300 К а=5-10 См/м, а для германия а=2,5 См/м, что в 10 —10 раз превышает проводимость диэлектриков и в то же время в 10 —10 раз уступает проводимости металлов. Зависимость о Т) полупроводников лишь в исключительных случаях и в небольшом температурном интервале может носить металлический характер как правило, и в полупроводниках, и в диэлектриках температурные зависимости проводимости подобны. Ширина энергетической щели в германии равна 0,72 эВ, а в кремнии 1,12 эВ, в то время как в алмазе — диэлектрике е такой же кристаллической структурой — запрещенная зона равна 7 эВ. Таким образом, с точки зрения зонной теории полупроводники принципиально отличаются от металлов наличием энергетической щели, в то время ак между полупроводниками и диэлектриками есть только количественное отличие.
Считается, что при Д [c.16]

Ковалентные кристаллы полупроводников (типа кремния) в отличие от ионных кристаллов — диэлектриков —прозрачны в инфракрасной области спектра, так как энергия квантов этой частоты недостаточна для возбуждения свободных электронов. Поэтому кремний и германий па частотах 10″—10 Гц используются как весьма совершенные и прозрачные материалы оптических элементов инфракрасной техники. Следовательно, эти типичные полупроводники в определенном частотном диапазоне играют роль весьма совершенных диэлектриков, в то время как обычно применяемые в оптике стекла и ионные кристаллы в инфракрасной области сильно отражают и поглощают электромагнитные волны (в этом диапазоне находятся собственные частоты колебаний кристаллической решетки).  

[c.17]

По современным представлениям электроны и дырки в кристаллах являются квантовыми возбужденными состояниями с отрицательным (—е) и положительным ( + е) зарядами соответственно. Важно отметить, что масса электрона или дырки в кристалле может существенно отличаться от массы т,, электрона в вакууме и, более того, зависит от направления движения электронов или дырок, являясь анизотропной (тензорной) величиной.

Поэтому при описании электронного механизма электропроводности диэлектриков и полупроводников вводится представление об эффективной массе Шэф.  [c.44]

Кристаллы и поликристаллы — важнейшие материалы электронной техники именно они используются во многих современных акустических, электронных и оптических приборах (см. гл. 5—7). В отличие от типичных полупроводников, в которых преобладает ковалентная связь атомов, кристаллические диэлектрики, в том числе пьезо-, пиро- и сегнетоэлектрики, характеризуются главным образом конной связью (хотя во многих случаях в них нельзя пренебрегать и другими видами связей [9]).  

[c.82]

Методы, основанные на комбинационном рассеянии света, эллипсометрии и тепловом расширении дифракционной решетки (естественной или искусственной), также значительно уступают интерференционной термометрии по чувствительности и помехозащищенности. По чувствительности ЛИТ полупроводников и диэлектриков на 2-ь4 порядка превосходит другие методы, основанные на регистрации отраженного, проходящего или рассеянного света. Выбор толщины пластинки и длины волны зондирующего света позволяет в пределах нескольких порядков изменять температурную чувствительность. Это свойство обусловлено двухступенчатым преобразованием изменений температуры в изменения интенсивности отраженного света. Такая схема позволяет управлять усилением преобразования, в отличие от многих методов, где преобразование является одноступенчатым, т. е. отражает только определенное свойство материала и не допускает усиления или ослабления коэффициента преобразования путем выбора условий считывания.  [c.175]

Полупроводники по удельному сопротивлению, которое при комнатной температуре составляет 10 — 10 Ом -м, занимают промежуточное положение между металлами и диэлектриками. Они обладают совокупностью специфических свойств, которые и выделяют их среди других веществ. В отличие от металлов полупроводники имеют в большом интервале температур отрицательный температурный коэффициент удельного сопротивления ТКр, т. е. положительный температурный коэффициент удельной проводимости ТКу (рис. И.1).  [c.47]


Удельное сопротивление серебра равно 1,62 X X 10″ Ом-м, меди 1,72-10″ Ом-м. Удельное сопротивление обычных металлов в любом случае менее 10 Ом-м имеет положительный температурный коэффициент, равный около 4-10-3 °С» . В сравнении с ними диэлектрики в большинстве своем характеризуются удельным сопротивлением 10 —Ю Ом-м. Удельное сопротивление проводников более чем на 15 порядков отличается от удельного сопротивления диэлектриков. Полупроводники имеют удельное сопротивление  [c.308]

Ом-м и занимают промежуточное положение между диэлектриками и проводниками. Температурный коэффициент сопротивления диэлектриков и полупроводников в отличие от металлов отрицателен. Различия между проводниками, и полупроводниками и диэлектриками объясняются с помощью зонной теории.  [c.308]

Лазерный отжиг — процесс восстановления кристаллической структуры твердого тела, нарушенной радиационным воздействием. В отличие от обычного, он позволяет контролировать температуру и время нагрева поверхностных слоев различных материалов на заданную глубину. Лазерный отжиг применяется для полупроводников, диэлектриков, металлов и сплавов. Его особенность состоит в том, что, во-первых, ввиду малой глубины проникновения лазерного излучения (10 +10″ см) не происходит нарушений более глубоких слоев во-вторых, время действия лазерного излучения при импульсном облучении может быть чрезвычайно малым (нано- и пикосекундный диапазон).  [c.523]

В отличие от проводников электропроводность полупроводников, как правило, быстро возрастает с увеличением температуры, а удельное сопротивление соответствен о падает. Уменьшение удельного электросопротивления полупроводников при повышении температуры может достигать нескольких тысяч раз. При очень низких температурах полупроводники превращаются в диэлектрики (изоляторы).  [c.10]

Полупроводники. К полупроводникам относится широкий круг конструкционных материалов, электрическая проводимость которых ограничивается интервалом от 10 ° до 10 Ом см т. е. меньше, чем у металлов и больше, чем у диэлектриков. Б отличие от металлов и диэлектриков для полупроводников характер-  [c.183]

Полупроводники качественно отличаются от металлов природой химических связей, структурой и физико-механическими свойствами. От диэлектриков полупроводники отличаются лишь количественно. Полупроводники — это вещества, имеющие при нормальной температуре удельную проводимость в интервале 10″ —10 Ом» м , которая зависит от вида и количества примесей, структуры вещества и внешних условий температуры, давления, электрических и магнитных полей, освещения, облучения ядерными частицами. В соответствии с зонной теорией у металлов валентные электроны легко переходят на уровни зоны проводимости и все валентные электроны участвуют в создании тока. У полупроводника энергетическая зона валентных электронов занята полностью и отделена от зоны проводимости зоной запрещенных энергий. К полупроводникам относятся вещества, для которых запрещенная зона равна (0,16- -5,1) 10″ Дж. Вещества с большей шириной запрещенной зоны относятся к диэлектрикам. Основу полупроводникового прибора составляет кристалл полупроводникового материала с одним пли несколькими электронно-дырочными р—м-переходами, которые получают,, вводя разнообразные примеси в различные участки одного и того же кристалла.  [c.230]

ПОЛЯРИТОН, составная квазичастица, возникающая нри вз-ствии экситона или онтич. фонона с фотонами частоты (й=81п>, где 8 — энергия экситона или фонона. Свойства П., напр, их дисперсии закон, существенно отличаются от свойств как экситонов, так и фотонов. П. обусловливают особенности оптич. спектров полупроводников и диэлектриков в области экситонных или фононных полос поглощения.  [c.578]

ОТЛИЧИЕ СВОЙСТВ ДИЭЛЕКТРИКОВ ОТ СВОЙСТВ МЕТАЛЛОВ И ПОЛУПРОВОДНИКОВ  [c.9]

Твердые тела, которые являются диэлектриками при Т = О, но имеют такие энергетические щели, что тепловое возбуждение при температурах ниже точки плавления может обусловливать заметную проводимость, называются полупроводниками. Ясно, что не существует четкого различия между полупроводником и диэлектриком грубо говоря, в наиболее важных полупроводниках энергетическая щель обычно меньше 2 эВ, а часто составляет лишь несколько десятых электронвольта. Типичные удельные сопротивления полупроводников при комнатной температуре лежат в интервале между 10 и 10 Ом-см (в отличие как от металлов, где р 10 Ом-см, так и от хороших диэлектриков, у которых р может достигать 10 Ом-см).  [c.185]

Все отличие проводников и полупроводников от диэлектриков с точки зрения теории Максвелла заключается в ненулевой величине слагаемого, содержащего плотность тока проводимости в уравнении  [c.192]

Зонная структура твердого тела является результатом взаимодействия волновой функции электрона с рещеткой. Зонная структура позволяет найти частоты и направления, для которых волновая функция электрона может или не может проходить через решетку. Отражение электронной волны под углами Брэгга от кристаллографических плоскостей является идеально упругим и не вносит вклада в электрическое сопротивление. Для каждого кристалла и каждой электронной конфигурации условия Брэгга налагают определенные ограничения на направление волнового вектора и значения энергий, которые может принимать электронная волна. Эти ограничения в направлениях и значениях энергий приводят к появлению щелей в почти непрерывном спектре энергий и направлений. Именно эти щели (порядка 1 эВ для полупроводников и 5 эВ или больше для хороших диэлектриков) обусловливают сильнейшие различия между металлами, полупроводниками и диэлектриками (рис. 5.2). Для металлов характерно, что уровень Ферми оказывается внутри зоны, имеющей вакантные энергетические уровни. Полупроводники имеют полностью заполненную разрешенную зону. Ширина запрещенной зоны у них невелика, н поэтому ие большое число электронов при тепловом возбуждении может перейти в расположенную выше разрешенную зону. Диэлектрик отличается от полупроводника тем, что его запрещенная зона очень велика, и практически ни один возбужденный электрон не может ее преодолеть.  [c. 190]


Первые попытки применения квантово-механической теории энергетического состояния электронов в диэлектриках и полупроводниках к интерпретации фотохимических и фотоэлектрических явлений в щелочно-галоидных кристаллах принадлежат П. С. Тар-таковскому [71]. На основе имевшихся в то время экспериментальных данных и общих соображений об энергетических уровнях в кристаллах Тартаковским впервые была построена схема энергетических уровней для ряда щелочно-галоидных соединений с учетом локальных электронных состояний различных центров окраски. Анализируя электронные переходы между различными уровнями энергии кристалла, можно было объяснить ряд оптических и фотоэлектрических свойств окрашенных кристаллов ще-лочно-галоидных соединений с единой точки зрения. Однако в отличие от полупроводников, для которых свет в области их фундаментального поглощения является фотоэлектрически активным, в щелочно-галоидных кристаллах не наблюдается внутреннего фотоэффекта под действием света в области первой полосы собственного поглощения. По этой причине попытки применения зонной теории к толкованию всей совокупности явлений, связанных с собственным поглощением, фотопроводимостью и люминесценцией щелочно-галоидных кристаллов наталкивались на существенные затруднения. Некоторые фундаментальные экспериментальные факты относительно свойств окрашенных щелочно-галоидных кристаллов не получили объяснения ни в энергетической схеме Тарта-ковского, ни в подобных более всеобъемлющих схемах, предлагавшихся позднее. В частности, оставалась совершенно непонятной сама возможность образования в кристалле столь устойчивой окраски под действием света или рентгеновых лучей, какая в действительности наблюдается у щелочно-галоидных кристаллов. В самом деле, при образовании в процессе фотохимического окрашивания свободных электронов, локализующихся затем на уровнях захвата, в верхней зоне заполненных уровней энергии должны образоваться свободные положительные дырки. Вследствие диффузии этих дырок в верхней зоне заполненных уровней вероятность их рекомбинации с электронами, локализованными в центрах окраски, должна быть достаточной, чтобы кристалл быстро обесцветился даже в темноте. Между тем, известно, что окраска кристалла весьма устойчива и сохраняется в темноте очень продолжительное время. Возможность локализации положительных дырок в предлагавшихся квантово-механических моделях не рассматривалась.  [c.30]

Электропроводность диэлектриков в отличие от полупроводников чаще всего носит не электронный, а ионный характер. Это связано с тем, что ширина запрещенной зоны в диэлектриках AW kT и лишь ничтожное количество электронов может отрываться от своих атомов за счет теплового движения. Ионы же часто оказываются слабо связанными в узлах решетки, и энергия W, необходимая для их срыва, сравнима с кТ, Например, в кристалле Na l = 6 эВ, а энергия  [c.123]

ЭЛЕКТРИЧЕСКИЙ ПРОБОЙ диэлектриков и полупроводников— резкое падение их электрич. сопротивления при достаточно высоком приложенном к образцу напряжении (см. также Пробой электрический). Э. п. отличается от теплового пробоя тем, что на подготовит, стадии пробоя ни разогрев, ни хим. процессы не имеют существенного значения, а также малым временем развития пробоя, слабой зависимостью пробивного напряжения от темп-ры. Э. п. обусловлен ударной ионизацией атомов и молекул электронами. Электрон получает возможность ударной ионизации, если энергия U, передаваемая ему электрич. полем, оказывается больше энергии U, теряемой электроном при рассеянии на фононах, дефектах и примесях кристаллич. рещётки. При этом электрон мо-  [c.514]

В качестве такого метода применяется сильнополевая туннельная ин-жекция заряда в диэлектрик, проводимая в режиме постоянного тока. В отличие от лавинной, режимы туннельной инжекции не зависят от характеристик области пространственного заряда полупроводника и определяются только параметрами границ раздела и самого диэлектрика. Использование туннельной инжекции позволяет точно дозировать ин-жекционную нагрузку структур и не требует создания специальных структур с инжекторами, т.е. она может проводиться в процессе формирования подзатворного диэлектрика до проведения металлизации.[c.124]

Диэлектриками являются неионизованные газы, а также жидкости и твердые тела, характеризующиеся полностью заполненной электронами валентной зоной и полностью свободной зоной проводимости. Если термического возбуждения электронов на уровни зоны проводимости не происходит, то такие вещества ведут себя как изоляторы. При малой энергетической щели Д Е или при большей температуре эти вещества ведут себя как полупроводники. Диэлектрики и полупроводники, в отличие от металлических проводников, экспоненциально уменьшают объемное сопротивление при повышении температуры.  [c.320]

Ограничения методов ЛТ. Степень универсальности метода определяется количеством разнородных объектов, для которых возможна регистрация температурно-зависимого параметра и термометрия. Методы ЛТ являются узкоспециализированными, в отличие от универсального метода термометрии по тепловому излучению. Узрсая специализация методов ЛТ означает, что любой из них позволяет проводить измерения лишь для ограниченного набора материалов, а в некоторых случаях имеются еще дополнительные требования к геометрической форме образца и свойствам поверхности. Например, для применения метода лазерной интерференционной термометрии полупроводников и диэлектриков необходимо, чтобы образец имел форму плоскопараллельной пластины, которая прозрачна для зондирующего излучения и имеет достаточно гладкие поверхности (тогда пластина может выполнять роль интерферометра Фабри-Перо). Компенсировать узкую специализацию рсаждого из методов ЛТ удается их многочисленностью и разнообразием.  [c.201]

Полевые транзисторы в отличие от биполярных имеют большее входное сопротивление, обладают значительно большей стабильностью при изменении температуры, создают меньшего уровня шум, обладают более высокой стойкостью к действию ионизирующего излучения. Разновидностью полевых транзисторов являются транзисторы с изолированным затвором, или МДП-транзисторы (металл — диэлектрик — полупроводник) или МОП-транзисторы (металл — оксид — полупроводник). Различают МДП-транзисторы с собственным каналом, характеристики которого представлены на рис. 3.23, и МДП-транзисторы с индуцированным каналом, характеристики которого даны на рис. 3.24. Параметры МДП-транзисторов аналогичны параметрам полевых транзисторов, транзисхоры имеют те же преимущества, что и биполярные. По сравнению с полевыми транзисторами МДП-транзисторы имеют большее входное сопротивление, достигающее 10 …10 Ом, и меньшую входную ёмкость, что позволяет их использовать на частотах до сотен мегагерц.  [c.472]

Сущность внутреннего фотоэффекта состоит в том, что при освещении полупроводников и диэлектриков от некоторых атомов отрываются электроны, которые, однако (в отличие от внешне1 о фотоэффекта), не выходят через поверхность тела, а остаются внутри него. В результате внутреннего фотоэффекта сопротивление полупроводников и диэлектриков уменьшается.  [c.165]

В 1931 англ. физик А. Вильсон указал на то, что существование Т. т. с различными электрич. св-вами связано с хар-ром заполнения эл-нами энергетич. зон при Г=ОК. Если все зоны либо целиком заполнены эл-нами, либо пусты, то такие тела не проводят электрич. ток, т. е. являются диэлектриками (рис. 3, а). Т. т., имеющие зоны, частично заполненные эл-нами,— металлы (рис. 3, б). Полупроводники отличаются от диэлектриков малой шириной запрещённой зоны между последней заполненной (валентной) зоно11 и первой пустой. зоной (зоной проводимости, рис. 3, в).  [c.736]


В полупроводниках и диэлектриках порог Ф. э. /1СОо= д+Х, где а— ширина запрещённой зоны, % — сродство к электрону, представляет собой высоту потенц. барьера для электронов проводимости (рис. 1, б). В не сильно легированных ПП эл-нов проводимости мало, поэтому здесь, в отличие от металлов, рассеяние энергии фотоэлектронов на эл-нах проводимости роли не играет. В этих материалах фотоэлектрон теряет энергию при вз-ствии с эл-нами валентной зоны (ударная ионизация) или с тепловыми колебаниями кристаллической решётки (рождение фононов). Скорость рассеяния энергии и глубина.  [c. 830]

Полупроводниковые твердые тела >, содержащие слабо связанные электроны, по величине электропроводности занимают промежуточное положение между металлами — хорошими проводниками тепла и электричества и дизлентриками — плохими проводниками тепла и электричества. Чистые полупроводники обладают смешанной (электронной и дырочной) проводимостью. С повышением температуры число свободных электронов увеличивается, в соответствии с этим увеличивается и доля электронной проводимости. При достаточно низких температурах все полупроводники становятся диэлектриками. В этом случае теплопроводность обусловливается главным образом упругими колебаниями решетки. Поэтому отличие полупроводников от диэлектриков носит скорее количественный, чем качественный характер.  [c.9]

ПОЛУПРОВОДНИКЙ — широкий класс веществ, в к-рых концентрация подвижных носителей заряда значительно ниже, чем концентрация атомов, в может изменяться под влиянием теми-ры, освещения иля относительно малого кол-ва примесей. Эти свойства, а также увеличение проводимости с ростом темп-ры, качественно отличают П. от металлов. Различие между П. и диэлектриками носит условный характер, к диэлектрикам обычно относят вещества, уд. сопротивление р к-рых при комнатной темп-ре (7″ = 300 К) >104—10 Ом-см.  [c.35]


Глава 19. Проводники и диэлектрики в электрическом поле

В школьном курсе физики есть раздел, посвященный электрическим свойствам проводников и диэлектриков и их поведению во внешнем электрическом поле. В необходимый минимум знаний по этому вопросу входит понимание явления электростатической индукции и его механизмов в проводниках и диэлектриках, а также умение находить в простейших ситуациях индуцированные в проводниках и диэлектриках заряды. Кратко рассмотрим эти вопросы.

В состав атомов входят заряженные частицы (электроны и протоны). Поэтому любое тело содержит огромное количество зарядов. Число протонов и число электронов в составе незаряженного тела одинаково, заряженное тело содержит разные количества протонов и электронов.

В зависимости от того, являются ли заряды внутри тела свободными или связанными, все вещества делятся на проводники, диэлектрики (или изоляторы) и полупроводники. В проводниках электрические заряды могут свободно перемещаться, и потому такие тела проводят электрический ток. К проводникам относятся все металлы, в которых носителями заряда являются «оторвавшиеся» от атомов валентные электроны (свободные электроны), а также растворы электролитов (кислот, щелочей и солей), в которых перемещаются положительные и отрицательные ионы.

В диэлектриках все заряды «привязаны» к покоящимся атомам и не могут перемещаться. Поэтому диэлектрики не проводят электрический ток. К диэлектрикам, например, относятся: газы, пластмассы, эбонит, резина, дистиллированная вода.

Вещества, занимающие по своей проводимости промежуточное положение между проводниками и диэлектриками, называются полупроводниками. Типичными полупроводниками являются кристаллические германий и кремний. В полупроводниках свободные носители заряда есть, но их мало. Не следует, однако, думать, что полупроводники являются просто «плохими» проводниками или «плохими» изоляторами. Промежуточная проводимость полупроводников приводит ко многим необычным их свойствам, которые отличают полупроводники как от проводников, так и от диэлектриков. С этими свойствами связаны многие применения полупроводников в технике.

При помещении проводника в электрическое поле свободные носители заряда внутри проводника перемещаются и на его поверхности образуются области положительного и отрицательного заряда. Такое явление разделения зарядов в проводнике под действием внешнего электрического поля называется электростатической индукцией или поляризацией проводника. В результате поляризации электрическое поле в пространстве изменяется и становится равным сумме внешнего поля и поля индуцированных зарядов. Можно доказать, что перемещение зарядов в проводнике будет происходить до тех пор, пока суммарное поле внутри проводника не станет равным нулю, а на его поверхности — перпендикулярным поверхности.

Такое свойство проводника позволяет находить индуцированные на его поверхности заряды. Для этого нужно ввести эти заряды как некоторые неизвестные величины, затем найти поле, создаваемое этими зарядами и суммарное поле, равное векторной сумме внешнего поля и поля индуцированных зарядов, приравнять суммарное поле внутри проводника к нулю. Решение полученного уравнения и позволит найти индуцированные заряды.

В диэлектрике поляризация также происходит, однако механизмы этого явления — другие. Как правило, молекулы диэлектрика являются полярными, т.е. какая-то область молекулы заряжена положительно, какая-то — отрицательно. При помещении диэлектрика во внешнее поле молекулы поворачиваются, и на определенные участки поверхности диэлектрика «выходят» своими положительными областями, на другие — отрицательными. В результате на поверхности диэлектрика образуются области положительного и отрицательного заряда, но при разрезании диэлектрика (в отличие от разрезания проводника) получившиеся части будут незаряженными. Благодаря поляризации диэлектрика поле в нем ослабляется, но не становится равным нулю. Характеристика диэлектрика , которая показывает, во сколько раз ослабляется поле в нем, называется диэлектрической проницаемостью.

Рассмотрим в рамках данного фактического материала задачи первой части.

В задаче 19.1.1 из нижеперечисленного списка веществ проводником электрического тока является металл — свинец (ответ 3).

В задаче 19.1.2 диэлектриком является мел (ответ 1; алюминий и железо — металлы, т.е. проводники тока, в водопроводной воде растворены различные соли в таком количестве, что она является прекрасным проводником электрического тока).

Как отмечалось ранее, при внесении металлического тела в электрической поле (задача 19.1.3) на поверхности тела индуцируются электрические заряды, сумма которых равна нулю. Все остальные предложенные ответы неверны: для приобретения электрического заряда телу нужно сообщить или забрать у него электроны, заряды не могут индуцироваться в объеме проводника — их невозможно там удержать.

Взаимодействие между зарядом и незаряженным диэлектрическим телом возникает (задача 19.1.4), причем это взаимодействие –— притяжение (ответ 2). Это взаимодействие возникает благодаря поляризации: из-за ориентации молекул диэлектрика часть поверхности тела, обращенная к заряду, приобретает заряд противоположного знака, дальняя от заряда часть поверхности тела — заряд того же знака (см. рисунок).

Поэтому возникнет две силы — притяжение близких участков и отталкивание дальних. Но поскольку индуцированные заряды — одинаковы по величине, а кулоновское взаимодействие убывает с ростом расстояния, притяжение сильнее отталкивания, и тело будет притягиваться к заряду.

Как указывалось во введении к настоящей главе, части металлического тела, внесенного в электрическое поле и разрезанного там (задача 19.1.5) будут заряжены. Поскольку направление вектора напряженности совпадает с направлением силы, действующей на положительный заряд, часть будет заряжена положительно, часть — отрицательно (ответ 2). Если тело является диэлектриком, то его части будут незаряженными (задача 19.1.6 — ответ 1).

После соединения проводником (задача 19.1.9) два металлических тела и соединяющий проводник будут представлять собой единое проводящее тело. Поэтому потенциалы любых точек этого тела должны быть одинаковы. Следовательно, выровняются потенциалы сфер (ответ 1).

В задачах с заземлением (задача 19.1.10) рассматривается следующая модель Земли: это проводящий шар с размерами, много большими размеров любых тел, имеющихся в задаче. Поэтому для потенциала Земли можно использовать формулу (18.8), которая для любых зарядов, с которыми мы имеем дело, дает нулевой результат. Поэтому при заземлении тела его потенциал становится равным нулю (ответ 2).

Сила взаимодействия противоположных электрических зарядов при внесении между ними диэлектрической пластинки (задача 19.2.1) увеличится (ответ 2). Действительно, в поле зарядов на поверхности пластинки будут индуцироваться заряды: ближе к положительному — минусы, ближе к отрицательному — плюсы (см. рисунок). В результате на каждый точечный наряду с той же самой силой притяжения к другому заряду (а она, конечно, не меняется, ведь принцип суперпозиции говорит о том, что все заряды взаимодействуют независимо) будут действовать две дополнительные силы. Это будет сила притяжения к зарядам того же знака и отталкивания от зарядов противоположного. А поскольку заряды противоположного знака ближе, сила притяжения будет больше. Возникновение дополнительной силы, направленной к пластинке, будет восприниматься как увеличение силы притяжения.

Как отмечалось выше (задача 19.1.7) потенциал электрического поля во всех точках проводящего тела одинаков. Поэтому можно ввести понятие потенциала проводящего тела, который определяется как потенциал электрического поля в любой точке этого тела. Поэтому для потенциала металлического шара из задачи 19.2.2 имеем , где , — заряд шара, — его радиус. Потенциал поля шара на расстоянии двух радиусов от его поверхности и, следовательно, трех радиусов от центра шара равен , т.е. одной трети от потенциала шара. Отсюда находим В (ответ 2).

Потенциал каждой капли ртути (задача 19.2.3) равен , где , — заряд капли, — ее радиус. После слияния заряд большой капли равен , а радиус , где — число капель (последнее следует из того, что объем большой капли равен сумме объемов капель). Отсюда находим потенциал большой капли

(ответ 2).

Поскольку после соединения шары будут представлять собой единое металлическое тело (задача 19.2.4), то заряд разделится между ними так, что потенциалы шаров будут одинаковы. Поэтому для зарядов шаров и выполнено условие

Отсюда находим (ответ 4).

Согласно принципу суперпозиции потенциал каждой точки складывается из потенциала, создаваемого в этой точке всеми зарядами. Поэтому потенциалы и внутренней и внешней сферы (задача 19.2.5) создаются зарядами внутренней и внешней сфер. А поскольку потенциал в любой точке внутри сферы определяется ее радиусом сферы (см. (18.8)), получаем

Аналогично находим потенциал внешней сферы

Отсюда находим

(ответ 3).

Чтобы найти разность потенциалов между двумя проводниками нужно мысленно перенести пробный заряд с одного из них на другой, найти работу, совершаемую электрическим полем при этом, разделить работу на величину пробного заряда. В задаче 19.2.6 между пластинками будет однородное поле с напряженностью . Поэтому работа поля над пробным зарядом при его перемещении с одной пластинки на другую есть . С другой стороны, работа поля следующим образом связана с разностью потенциалов . Отсюда находим разность потенциалов пластин

(ответ 3).

Поскольку напряженность поля между двумя параллельными пластинками, заряженными одинаковым зарядом равна нулю (см. задачу 18.2.8), то при перенесении пробного заряда с одной пластины на другую поле не совершает работу. Следовательно, разность потенциалов между такими пластинками в задаче 19.2.7 равна нулю (ответ 4).

В задаче 19.2.8 заряды распределятся только по внешней поверхности полого шара (если бы весь заряд или какая-то его часть находилась на внутренней поверхности, то в объеме проводника было бы электрическое поле, чего быть не должно). А поскольку заряд, расположенный на поверхности сферы, создает поле только снаружи этой сферы, то напряженность будет отлична от нуля только в области 3. Поэтому правильный ответ в задаче — 4.

В задаче 19.2.9 заряды индуцируются и на внешней и на внутренней поверхностях полого шара, причем их сумма равна нулю. Результирующее поле будет создаваться центральным зарядом и индуцированными зарядами, которые, фактически, представляют собой равномерно заряженные сферы. А поскольку поле сферы равно нулю внутри этой сферы, то суммарное поле в полости (в области 1) равно полю точечного заряда, т.е. не равно нулю. Внутри металлического тела (в области 2) поле равно нулю, как и внутри любого проводника. Снаружи шара поля индуцированных зарядов компенсируют друг друга, поэтому суммарное поле равно полю точечного заряда, т.е. не равно нулю. Поэтому правильный ответ в этой задаче — 2.

В задаче 19.2.10 на внешней и внутренней поверхности сферической оболочки будут индуцироваться такие заряды, что суммарное поле (внешнее плюс поле индуцированных зарядов) внутри оболочки будет равняться нулю. Пусть на внутренней поверхности будет индуцирован заряд — , тогда на внешней поверхности будет индуцирован заряд . Поле внутри оболочки (в области 2) будет создаваться только точечным зарядом и зарядами внутренней поверхности (заряд внешней поверхности благодаря ее сферичности в этой области электрического поля не создает). С другой стороны это поле равно нулю. Отсюда заключаем, что заряд внутренней поверхности оболочки противоположен по знаку центральному точечному заряду и равен ему по величине . Следовательно, заряд внешней поверхности оболочки центральному заряду (ответ 1).

Проводники и диэлектрики

Цели:

  • Образовательная: формирование представления о проводниках и диэлектриках; обеспечение в ходе урока понимания учащимися отличия проводников от диэлектриков с точки зрения электронной теории; создать условия для формирования понятие о диэлектриках и их физической природе с точки зрения электронной теории.
  • Развивающая: способствовать развитию познавательной активности, образного мышления; способствовать дальнейшему развитию умений выделять главное, сравнивать, анализировать, делать выводы.
  • Воспитательная: воспитание чувства ответственности и готовности к сотрудничеству; приобретение навыков общения и самоорганизации; способствовать формированию научного мировоззрения.

Оборудование: персональный компьютер, мультимедийный проектор, экран, электрометр с набором тел, гильза на штативе, пластина из оргстекла, лист пластика калориметр.

Методы обучения: объяснительно-иллюстративный, практический.

Новая тема:

Опыт. Зарядим электроскоп эбонитовой палочкой и прикоснёмся к электроскопу поочередно деревянной палочкой, пластмассовой линейкой, а затем металлическим стержнем.

Вывод. Все вещества по проводимости бывают двух видов проводники и диэлектрики.

Давайте выясним откуда берутся заряды у проводников. Все металлы являются хорошими проводниками электрического тока. Рассмотрим строение натрия. У натрия 11е, которые распределены по трем энергетическим уровням. Последний электрон слабо притягивается к ядру. Следовательно атом может потерять электрон и становится положительным ионом.

Проводники состоят из положительных заряженных ионов. Которые участвуют в тепловом движении и свободных электронов, которые перемещаются по всему проводнику.

Опыт. Незаряженная гильза и эбонитовая палочка. Разряженную эбонитовою палочку поднесем к гильзе. Гильза притянется к эбонитовой палочке. Под действием электрического поля электроны в гильзе приходят в движение и собираются на левой стороне гильзы, а правая сторона заряжается положительно. В внутри гильзы возникает свое поле, направленное против внешнего , следовательно электроны прекращают движение. Eвн. = E соб.

Рассмотрим проводник в электростатическом поле.

Вывод:

— внутри проводника поля нет

— весь заряд сосредоточен на поверхности проводника

Историческая справка. Опыт Майкл Фарадей. Сообщение ученика.

Опыт. Электроскоп, эбонитовая палочка, калориметр.

Вывод: внутри проводника электростатического поля нет.

Рассмотрим диэлектрик на примере поваренной соли NaCl и убедимся , что у диэлектриков нет свободных электронов.

Хлор захватывает электроны у натрия следовательно натрий заряжается положительно , хлор отрицательно. Получается система из двух разноименных зарядов, связанных между собой.

Электрический диполь – совокупность двух точечных зарядов, равных по модулю и противоположны по знаку.

Диэлектрики делятся на полярные и неполярные.

ПолярныеНеполярные
Поваренная соль

Спирт

Вода

Инертные газы

Бензол

Полиэтилен

Рассмотрим диэлектрик в электростатическом поле.

На каждый диполь со стороны поля действует пара сил. Под действием этих сил положительные диполи смещаются в направлении электрического поля,а отрицательные в противоположную сторону.

Смещение положительных и отрицательных связанных зарядов диэлектрика в противоположные стороны называют поляризацией.

Диполи создают свое электрическое поле Eсоб. Направленное против E вн.

Тепловое движение препятствует созданию упорядоченной ориентации всех диполей.

Под действием поля происходит частичная ориентация диполей.

Вывод: Собственное поле диэлектрика ослабляет внутри диэлектрика внешнее электрическое поле.

Для того чтобы описать, как сильно ослабляет диэлектрик электрическое поле, вводят величину, которую называют диэлектрической проницаемостью.

Если обозначить

Е.- напряжённость электрического поля в вакууме;

Е — напряжённость электрического поля в диэлектрике;

— диэлектрическая проницаемость среды, то получим формулу

Подведение итогов урока

Домашнее задание: параграфы 95, 96, 97.

2. Проводники, диэлектрики и поток электронов | 1. Основы электроники | Часть1

2. Проводники, диэлектрики и поток электронов

Проводники, диэлектрики и поток электронов

Электроны различных типов атомов обладают разными степенями свободы перемещения. В некоторых материалах, таких как металлы, внешние электроны атомов настолько слабо связаны с ядром, что легко могут покидать свои орбиты и хаотично двигаться в пространстве между соседними атомами даже при комнатной температуре. Такие электроны часто называют свободными электронами

В других типах материалов, таких как стекло, у электронов в атомах существует очень небольшая свобода перемещения. Однако внешние силы, например физическое трение, могут заставить некоторые из этих электронов покинуть собственные атомы и перейти к атомам другого материала, но они не могут свободно перемещаться между атомами  материала.

Эта относительная подвижность электронов в материале известна как электропроводность. Электропроводность определяется типами атомов материала (количество протонов в ядре атома, определяющее его химическую идентичность) и способом соединения атомов друг с другом. Материалы с высокой подвижностью электронов (много свободных электронов) называются проводниками, а материалы с низкой подвижностью электронов (мало или совсем нет свободных электронов) называются диэлектриками.

Ниже приведено несколько примеров наиболее распространенных проводников и диэлектриков:

Проводники:

  • серебро
  • медь
  • золото
  • алюминий
  • железо
  • сталь
  • латунь
  • бронза
  • ртуть
  • графит
  • грязная вода
  • бетон


Диэлектрики:

  • стекло
  • резина
  • нефть
  • асфальт
  • стекловолокно
  • фарфор
  • керамика
  • кварц
  • (сухой) хлопок
  • (сухая) бумага
  • (сухая) древесина
  • пластмасса
  • воздух
  • алмаз
  • чистая вода

Следует понимать, что не у всех проводящих материалов одинаковый уровень проводимости, и не все диэлектрики одинаково сопротивляются движению электронов. Электрическая проводимость аналогична прозрачности некоторых материалов: материалы, которые легко «пропускают» свет, называют «прозрачными», а те, которые его не пропускают, называют «непрозрачными». Однако, не все прозрачные материалы одинаково пропускают свет. Оконное стекло — лучше чем органическое стекло, и конечно лучше чем «прозрачное» стекловолокно. Так же и с электрическими проводниками, некоторые из них лучше пропускают электроны, а некоторые — хуже.

Например, серебро является лучшим проводником в представленном выше списке «проводников», обеспечивая более легкий проход электронов чем любой другой материал из этого списка. Грязная вода и бетон также значатся как проводники, но эти материалы являются существенно менее проводящими чем любой металл.

Некоторые материалы изменяют свои электрические свойства при различных температурных условиях. Например, стекло является очень хорошим диэлектриком при комнатной температуре, но становится проводником, если его нагреть до очень высокой температуре. Газы, такие как воздух, в обычном состоянии — диэлектрики, но они также становятся проводниками при нагревании до очень высоких температур. Большинство металлов, наоборот, становятся менее проводимыми при нагревании, и увеличивают свою проводимость при охлаждении. Многие проводники становятся идеально проводящими (сверхпроводимость) при экстремально низких температурах.

В обычном состоянии движение «свободных» электронов в проводнике хаотично, без определенного направления и скорости. Однако, путем внешнего воздействия можно заставить эти электроны двигаться скоординировано через проводящий материал. Такое направленное движение электронов мы называем электричеством, или электрическим током. Чтобы быть более точным, его можно назвать динамическим электричеством в отличие от статического электричества, в котором накопленный электрический заряд неподвижен. Электроны могут перемещаться в пустом пространстве внутри и между атомами проводника точно так же, как вода течет через пустоту трубы. Приведенная аналогия с водой в нашем случае уместна, потому что движение электронов через проводник часто упоминается как «поток».

Поскольку электроны двигаются через проводник равномерно, то каждый из них толкает находящиеся впереди электроны. В результате все электроны движутся одновременно. Начало движения и остановка электронного потока на всем протяжении проводника фактически мгновенны, даже несмотря на то, что движение каждого электрона может быть очень медленным. Приблизительную аналогию мы можем увидеть на примере трубки, заполненной мраморными шариками:


Трубка заполнена мраморными шариками точно также, как проводник заполнен свободными электронами, готовыми к перемещению под воздействием внешних факторов. Если вставить еще один мраморный шарик в эту заполненную трубку слева, то последний шарик сразу выйдет из нее справа. Несмотря на то, что каждый шарик прошел короткое расстояние, передача движения через трубку в целом произошла мгновенно от левого конца до правого, независимо от  длины трубки. В случае с электричеством, передача движения электронов от одного конца проводника к другому происходит со скоростью света: около 220 000 км. в секунду!!! Каждый отдельный электрон проходит через проводник в гораздо более медленном темпе.

Если мы хотим, чтобы электроны текли в определенном направлении к определенному месту, мы должны проложить для них соответствующий путь из проводов, точно так же, как водопроводчик должен проложить трубопровод, чтобы подвести воду к нужному месту. Для облегчения этой задачи, провода изготавливаются из хорошо проводящих металлов, таких как медь или алюминий.

Электроны могут течь только тогда, когда у них есть возможность перемещаться в пространстве между атомами материала. Это означает, что электрический ток может быть только там, где существует непрерывный путь из проводящего материала, обеспечивающего передвижение электронов. По аналогии с мраморными шариками мы можем видеть, что шарики будут «течь» через трубку только в том случае, если она будет открыта с правой стороны. Если трубку заблокировать, то мрамор будет «накапливаться» в ней, а соответственно не будет и «потока». То же самое верно и для электрического тока: непрерывный поток электронов требует непрерывного пути для обеспечения этого потока. Давайте посмотрим на схему, чтобы понять, как это работает:


 

Тонкая, сплошная линия (показанная выше) является схематическим обозначением непрерывной части провода. Так как провод сделан из проводящего материала, такого как медь, у  составляющих его атомов существует много свободных электронов, которые могут свободно перемещаться по нему. Однако, в пределах такого провода никогда не будет направленного и непрерывного потока электронов, если у него не будет места, откуда приходят электроны и места, куда они идут. Давайте в нашу схему добавим гипотетические  «Источник» и «Получатель» электронов:

 


Теперь, когда Источник поставляет новые электроны в провод, через этот провод пойдет поток электронов (как показано стрелками, слева-направо). Однако, поток будет прерван, если проводящий путь, образованный проводом, повредить:


 

В связи с тем, что воздух является диэлектриком, образовавшийся воздушный разрыв разделит провод на две части. Некогда непрерывный путь нарушается, и электроны не могут течь от Источника к Получателю. Аналогичная ситуация получится, если водопроводную трубу разрезать на две части, а концы в месте разреза закупорить: вода в этом случае течь не сможет. Когда провод был одним целым, у нас была электрическая цепь, и эта цепь была нарушена в момент повреждения. 

Если мы возьмем еще один провод  и соединим им две части поврежденного провода, то снова будем иметь непрерывный путь для потока электронов. Две точки на схеме показывают физический (металл-металл) контакт между проводами:


Теперь у нас снова есть цепь, состоящая из Источника, нового провода (соединяющего поврежденный) и Получателя электронов. Если рассматривать аналогию с водопроводом, то  установив тройник на одной из закупоренных туб, мы можем направить воду через новый сегмент трубы к месту назначения. Обратите внимание, что в правой части поврежденного провода нет потока электронов, потому что он больше не является частью пути от Источника до получателя электронов.  

Следует отметить что проводам, в отличие от водопроводных труб, которые в конечном итоге разъедаются ржавчиной, никакой «износ» от воздействия потока электронов не грозит. При движении электронов, в проводнике возникает определенная сила трения, которая может вырабатывать тепло. Подробнее эту тему мы рассмотрим несколько позже.

Краткий обзор:

  • В проводниках, электроны находящиеся на внешних орбитах атомов могут легко покинуть эти атомы, или наоборот присоединится к ним. Такие электроны называются свободными электронами.
  • В диэлектриках внешние электроны имеют намного меньше свободы передвижения, чем в проводниках.
  • Все металлы являются электрически проводящими.
  • Динамическое электричество, или электрический ток — это  направленное движение электронов через проводник.
  • Статическое электричество — это неподвижный (если на диэлектрике), накопленный заряд, сформированный избытком или недостатком электронов в объекте.
  • Для обеспечения потока электронов нужен целый, неповрежденный проводник, который обеспечит приём и выдачу электронов.

Источник: Lessons In Electric Circuits

Отличие полупроводников от металлов и диэлектриков

    Электропроводимость металлов выше Ю Ом -см , диэлектриков ниже 10 Ом -см- (при 298 К), проводимость полупроводников лежит между этими значениями. Однако главное отличие полупроводников от металлов состоит не в количественной оценке электропроводности, а в характере зависимости проводимости от температуры (рис. 4.19). Температурная зависимость проводимости металлов определяется временем свободного пробега электронов. С повышением температуры тепловые колебания атомов в узлах кристаллической решетки усиливаются, что приводит к увеличению взаимодействия их с электронами и к понижению проводимости. [c.187]
    Отличия полупроводников от металлов и диэлектриков [c.243]

    Важнейшее отличие металлов от полупроводников и диэлектриков. Электронная и дырочная проводимость. [c.231]

    В качестве иллюстрации того, как внешние факторы влияют на электрическую проводимость полупроводников, приведем следующие данные. Селеновый полупроводник при глубоком охлаждении становится изолятором его электрическая проводимость уменьшается в миллиарды раз (I млрд = Ю ). Вообще все полупроводники при температурах, близких 0° К, становятся диэлектриками. С повышением температуры у них электрическая проводимость повышается. В этом отношении полупроводники отличаются от металлов, у которых электрическая проводимость при нагревании понижается. [c.452]

    Иначе говоря, полупроводники и диэлектрики отличаются от металлов тем, что валентная зона у них заполнена электронами, а ближайшая свободная зона (зона проводимости) отделена от валентной зоной запрещенных состояний. При этом ширина запрещенной зоны у полупроводников от десятых долей до 3 эВ, а у диэлектриков от 3 до 5 эВ (рис, 72). [c.265]

    При повышении температуры проводимость полупроводников в отличие от металлов обычно возрастает (см. 2). Электропроводность диэлектриков тоже возрастает. При температуре, близкой к абсолютному нулю, проводимость полупроводников и диэлектриков практически нулевая. По электрическим свойствам полупроводники стоят ближе к диэлектрикам, чем к металлам, от которых они имеют принципиальное качественное отличие. [c.232]

    С позиций зонной теории, отличие металлов от полупроводников и диэлектриков в случае полностью заполненной валентной [c.49]

    Объяснение электропроводности металлов, полупроводников и диэлектриков дается на основе квантовой теории строения кристаллических тел — так называемой зонной теории. Рассмотрим некоторые общие положения этой теории. Переход атомных паров в кристаллическое вещество можно рассматривать как химическую реакцию, так как оптические, термодинамические, электрофизические и другие свойства твердых тел отличаются от свойств газов. Важно отметить, что атомные спектры газов имеют линейчатое строение, а спектры твердых тел имеют сплошной характер или полосатую, очень сложную структуру. Уже при взаимодействии двух одинаковых атомов дискретные атомные энергетические уровни расщепляются и превращаются в полосы. Тем большее расщепление уровней происходит, когда большое число N атомов, например лития, сближается с далеких расстояний до расстояний, на которых они находятся в кристаллической решетке. На рис. 70, а это расстояние между ядрами обозначено на оси абсцисс буквой о- По оси ординат отложена энергия. Находясь на больших расстояниях, атомы не взаимодействуют друг с другом, и диаграмма уровней будет такая же, как и для изолированного атома лития (1 25 ). При сближении атомов начнется взаимодействие между ними, прежде всего у каждого из них станет расщепляться уровень валентных электронов (2х). Уровень 2з) расщепляется в систему весьма близко расположенных N уровней, образуя целую полосу (зону) уровней. Более глубокие уровни при образовании кристалла оказываются совсем не расщепленными или только незначительно расщепленными. [c.233]


    Как указывалось выше, при повышении температуры наблюдается падение удельной электрической проводимости металла. В противоположность это.му проводимость полупроводников с повышением температуры растет (у диэлектриков это выражено слабее). Электрическая проводимость полупроводников в отличие от металлов не уменьшается, а увеличивается в присутствии небольших количеств примесей, при наличии дефектов в строении кристаллических решеток, а также под действием света и различного рода излучений. [c.265]

    В общем случае величина а Т. т. зависит от механизма рассеяния носителей заряда, к-рое может происходить на тепловых колебаниях атомов (ионов), нейтральных и заряженных собств. и примесных точечных дефектах, линейных, поверхностных и объемных дефектах кристаллич. решетки. В случае металлов а имеет электронную природу и подчиняется закону Ома. Для металлов характерно уменьшение а с т-рой. В отличие от металлов у полупроводников с повышением т-ры а увеличивается вследствие значит, возрастания концентрации своб. носителей заряда. В диэлектриках осн. носители заряда-ионы, вследствие чего а сопровождается переносом в-ва. Электронная проводимость диэлектриков возникает лишь при высоких электрич. напряжениях, близких к пороговым и соответствующих пробою. Как и в полупроводниках, о возрастает с повышением т-ры. [c.502]

    В отличие от металлов, у полупроводников все уровни в валентной зоне целиком заполнены и электропроводность может осуществляться лишь при переходе электронов в возбужденную зону (зону проводимости). Для перехода электронов с энергетических уровней валентной зоны в зону проводимости должна быть преодолена запрещенная зона, ширина которой может быть различной. Запрещенная зона может быть настолько широка, что при комнатных температурах приложенная извне тепловая энергия или, например, энергия освещения окажется недостаточной для перехода электронов в зону проводимости в количествах, обеспечивающих электропроводность вещества в указанных выше пределах. Такие твердые тела являются диэлектриками к ним обычно относят вещества, у которых ширина запрещенной зоны превышает 2 эв. [c.62]

    Все вещества по электропроводности можно разделить на три класса металлы, полупроводники, изоляторы (диэлектрики). Удельная электропроводность у металлов 6,3-10 -i- 4-10 сим м, у полупроводников 10 10 спм м, у диэлектриков 10 -i—i- 10 сим м. Таким образом, к полупроводникам относят вещества, электропроводность которых отличается между собой на много порядков. [c.265]

    Возникновение электрохимии полупроводников как новой главы теоретической электрохимии обусловлено двумя основными причинами. Во-первых, многие электрохимические процессы, протекающие на границе электрод — электролит, совершаются фактически на поверхности, обладающей полупроводниковыми свойствами, со всеми особенностями, присущими такого рода материалам. Проводимость этих поверхностных слоев — окислов металлов, их гидридов, интерметаллических соединений и т. п.— по своей величине лежит между проводимостью металлов и диэлектриков. Она чувствительна к внедрению в основной слой следов примесей и в противоположность металлам увеличивается с температурой. Прохождение тока через полупроводники в общем случае осуществляется электронами (п-проводимость) или дырками, т. е. вакансиями, оставшимися после ухода электронов в другую энергетическую зону (р-проводимость). В отличие от металлов, в полупроводниках вблизи их поверхности раздела с другими фазами имеется широкая область объемного заряда, что значительно усложняет картину двойного электрического слоя. Выяснение кинетики многих электрохимических реакций (процессы в химических источниках тока, анодное растворение металлов и т. п.) становится поэтому невозможным без разработки электрохимии полупроводников. Во-вторых, в самой технологии получения полупроводниковых материалов, идущих на изготовление радиотехнических приборов, солнечных батарей и т. п., важную роль играют процессы, являющиеся по своей природе электрохимическими. К ним относятся, например, анодное и обычное травление полупроводников, осаждение тонких слоев металла на поверхность полупроводников и др. [c.491]

    Все твердые вещества по их электрической проводимости можно разделить на три типа проводники, диэлектрики и полупроводники. Металлы проводят электрический ток очень хорошо, диэлектрики — очень плохо. Диэлектриками могут быть ковалентные вещества, состоящие из небольших молекул, например, трииодид фосфора, для которых энергия, необходимая для отрыва электрона от одной молекулы и передачи его другой, слишком велика для практических целей . Диэлектриками являются почти все ионные кристаллы, а также твердые вещества с непрерывной ковалентной решеткой, такие, как кварц или алмаз (но в отличие от алмаза графит — проводник). [c.140]

    В гл. 1 мы качественно рассмотрели природу сил, связывающих атомы и ионы в твердых телах, однако многие физические и химические свойства твердых тел можно понять только после более глубокого изучения электронного строения и природы сил связи в твердом теле. Одна из главных задач любой теории твердого тела состоит в том, чтобы объяснить, почему твердые тела могут принадлежать столь различным по свойствам классам, как диэлектрики и металлы (которые по электропроводности могут отличаться в 10 раз ). В первых теориях твердого тела рассматривалась модель свободного электрона, согласно которой валентные электроны могут свободно двигаться по всему объему твердого тела, У этой модели были определенные достоинства, но и существенные недостатки, и среди них отсутствие удовлетворительного объяснения факта существования диэлектриков и полупроводников, свойства которых удалось объяснить лишь позднее с позиций квантовой теории. Мы начнем с краткого знакомства с природой сил связи в молекулах, а затем перейдем к твердому телу. [c.30]


    Существуют три принципиальные структуры транзисторов на горячих электронах. Они отличаются в структуре эмиттера и механизме инжекции горячих электронов в металл базы. Это туннельно-эмиссионный транзистор, транзистор на эмиссии, ограниченной пространственным зарядом, и транзистор с эмиттером Шоттки. В нервом случае электроны инжектируются в металлическую базу через тонкий слой изолятора. Во втором случае горячие электроны инжектируются в диэлектрик и затем в металл базы. Поток электронов в этом случае определяется пространственным зарядом, образующимся в диэлектрике у инжектирующего контакта. В третьем случае горячие электроны инжектируются в металлическую базу выпрямляющим контактом металл — полупроводник. [c.72]

    У поглощающих сред коэффициент поглощения k существенно отличается от нуля, и в формулах (XII.1) показатель преломления становится комплексным [см. формулу (1.7)]. В видимой области спектра величина k колеблется у диэлектриков и полупроводников в пределах 10 -i-l, а у металлов—14-10. Выражения для коэффициентов отражения сильно усложняются и принимают вид  [c.217]

    У диэлектриков концентрация электронов проводимости при комнатной температуре на много порядков ниже, чем у металлов. И хотя с температурой концентрация, а с нею и электропроводность, быстро растет, абсолютные значения последней в области умеренно высоких температур еще очень малы. Диэлектрики скорее пригодны в этих интервалах температур к роли пассивных элементов схем в отличие от полупроводников, из которых создаются активные элементы. [c.13]

    Электронная проводимость связана с образованием электронов в полимерах при ионизации макромолекул, которая может быть вызвана нагреванием, радиационным или световым воздействием. Присутствие пигментов и других неорганических веществ в покрытии благоприятствует электронной проводимости. Электронная проводимость пленок кристаллических полимеров выше, чем аморфных, ионная — наоборот. Особенно высокой электронной проводимостью отличаются полимеры-полупроводники, а также композиции с углеродными и металлическими наполнителями (сажей, графитом,, порошками металлов). Изготовленные из них покрытия по электрической проводимости занимают промежуточное положение между диэлектриками и проводниками для них уу 10 — 10″1 См/м. Электрическая проводимость большинства лакокрасочных покрытий находится на уровне электрической проводимости полимеров и составляет 10 -10 См/м. [c.132]

    Не вдаваясь в подробности строения зон, подчеркнем, что полупроводники и диэлектрики отличаются от металлов тем, что валентная зона у них при Т 0° К всегда полностью заполнена электронами, а ближайитя свободная зона (зона проводимости) отделена от валентной зоной запрещенных состояний Ширина запрещенной зоны АБ у полупроводников — от десятых долей электрон-вольт до 3 эв (условно), а у диэлектриков — от 3 до 5 эв (условно) Если между полупроводниками и диэлектриками имеется только количественное различие, то отличие их от металлов качественное. Чтобы проходил ток в металле, не требуется никакого другого воздействия, кроме наложения электрического поля, так как валентная зона в металле не заполнена или перекрывается с зоной проводимости (рис. 71, а). [c.235]

    Дискретным уровням атома в твердом теле соответствует всегда дискретная система разрешенных зон, разделенных запрещенными зонами. Если электроны образуют в атомах или моле1жидкого вешества созда ются зоны с полностью заполненными уровнями, поэте му такие вещества при абсолютном нуле имеют свойства изоляторов. Сюда относятся решетки благородных газов, молекулярные и ионные решетки соединений с насыщенными связями. В решетках алмаза, кремния, германия, а-олова, соединений тяпа А» В , А В , Si каждый атом связан единичными ковалентными связями с четырьмя ближайпгими соседними, так что вокруг него образуется законченная группа электронов s p и валентная зона оказывается заполненной. Необходимо подчеркнуть, что полупроводники и диэлектрики отличаются от Металлов тем, что валентная зона у них при Гл О К всегда полностью заполнена электронами, а ближайшая свободная зона (зона проводимости) отделена от валентной зоной запрещенных состояний. Ширина запрещенной зоны АЕ у полупроводников — от десятых долей до 3 эВ (условно), а у диэлектриков — то 3 до 5 эВ (условно). Если между полупроводниками и диэлектриками имеется только количественное различие, то отличие их от металлов качественное. Чтобы проходил ток в металле, не требуется никакого другого воздействия, кроме наложения электрического поля, так как валентная зона в металле не заполнена или перекрывается с зоной проводимости (рис. 71, а). [c.292]

    Электрич. проводимость а Т. т. определяется в первую очередь характером заполнения электронами энергетич. зон (см. рис.). Т. т. с металлич. типом хим. связи (металлы) характеризуются высокой степенью обобществления валентных электронов (электронов проводимости), перекрыванием разрешенных энергетич. зон и частичным заполнением разрешенных зон электронами. Такие Т. т. являются хорошими проводниками. Б отличие от них полупроводники и диэлектрики при Г= О К имеют полностью заполненные лргбо пустые, неперекрывающиеся, разрешенные зоны. Для диэлектриков характерны большие значения ширины запрещенной зоны АЕ между валентной (заполненной) и незаполненной зоной (зоной проводимости), вследствие чего в обычных условиях они практически не содержат своб. электронов и ие проводят электрич. ток. Полупроводники, Гринщшиально не отличаясь от диэлектриков по зонному [c.502]

    Полупроводники — вещества, по электропроводности промежуточные между проводниками и диэлектриками (изоляторами). Их электропроводность зависит от температуры, увеличиваясь при ее повышении (отличие от металлов), от количества и природы примесей, воздействия электрического поля, света и других внешних факторов, К П. принадлежат бор, углерод (алмаз), кремний, германий, олово (серое), селен и теллур, карбид кремния Si соединения типа (индий — сурьма, индий — мышьяк, галлий — сурьма, алюминий — сурьма), соединения двух или трех элементов, в состав которых входит хотя бы один из элементов IV—VH групп периодич. системы Д. И. Менделеева, органические вещества (полицены, азоарома-тические соединения, фталоцианины, некоторые свободные радикалы и др.). К чистоте полупроводниковых материалов предъявляют повышенные требования напр., в германии контролируют содержание примесей 40 эле.ментов, в кремнии — 27 элементов. Содержание примесей не должно превышать 10 — 10- %. П. имеют большое практическое значение. [c.107]

    Ширина запрещенной зоны для разных веществ различна для Si, Ge, GaAs и Na l она составляет соответственно 1,21 0,75 1,45 и 7,0 эВ. Принято считать полупроводниками вещества с шириной запрещенной зоны менее 1,5 эВ. В металлах запрещенная зона отсутствует (принято считать, что зона проводимости и валентная зона перекрываются), а в диэлектриках превышает 1,5 эВ. Если ширина запрещенной зоны невелика, то в результате теплового движения электроны могут преодолевать ее. Поэтому электропроводность полупроводников, в отличие от металлов, круто растет с повышением температуры. Переход через зону возможен также и при поглощении кванта энергии, чем объясняется так называемый внутренний фотоэффект, т. е. резкое увеличение проводимости под действием излучения. [c.295]

    Свободная валентность в алюмосиликате как диэлектрике в отличие от металлов и полупроводников, где она, по Волькенштейну, может свободно неремеш аться па новерхности либо быть локализованной на ее структурных дефектах, имеет ряд специфических свойств  [c.377]

    В отличие от гомогенного для гетерогенного катализа нет единой теории, позволяющей описать все наблюдаемые явления. Особенность гетерогенных каталитических реакций заключается в образовании на твердой поверхности катализатора хемосорбирован-ных (на активных центрах) комплексов, которые не способны существовать индивидуально и не могут быть названы промежуточными соединениями. Хемосорбционные комплексы одного из реагентов в дальнейшем вступают во взаимодействие с компонентами реакционной смеси, образуя продукты реакции и освобождая активные центры поверхности. Характер взаимодействия в значительной мере зависит от электронной структуры твердого катализатора. С этой точки зрения активные металлы с их легкоподвижиыми электронами обычно склонны к образованию относительно прочных поверхностных комплексов и поэтому каталитически малоактивны. Диэлектрики с ничтожно малой концентрацией свободных электронов плохо образуют поверхностные комплексы и потому также не отличаются каталитической активностью. А на поверхности полупроводников и малоактивных металлов, которые характеризуются промежуточными значениями электронной концентрации, хорошо образуются метастабильные ассоциаты, чем и определяется их высокая каталитическая активность. Эти представления позволяют связать каталитические свойства полупроводников с другими их параметрами электрической проводимостью, энергией активации электрической проводимости, особым состоянием поверхности и т. и. Так, например, промотирование сульфатами щелочных [c.236]


9Проводники и диэлектрики в электрическом поле._Разработка урока

I.                    Организационный момент. 

II.                 Постановка темы, целей урока и критериевдостижения успеха.

  1. Активизация  мышления учащихся.

1.      Как называют вещества, проводящие и не проводящие электрический ток?

2.      Как вы думаете, почему диэлектрики не проводят электрический ток?

3.      Диэлектрики иначе называются изоляторами, назовите примеры твердых тел, являющихся диэлектриками (изоляторами).

4.      Где используют изоляторы?

5.      Что относится к проводникам электрического тока?

Эксперимент1. Зарядим электрометр и прикоснёмся к нему поочерёдно сначала пластмассовым стержнем, а затем металлическим стержнем. Что вы видите? (Во-втором случае, электрометр разрядился.Следовательно, металл проводит заряды, а пластмасса нет).

Изучение нового материала. Тема урока: Проводники и диэлектрики  в электрическом поле.

Работа в группах. Составление характеристик проводников и диэлектриков. Постерная защита.

Задание 1.  Группа 1. Проводники в электрическом поле.

1.      Объясните, почему проводники проводят электрический ток. Откуда  берутся заряды в проводниках? Объяснить на примере  строения металла.

2.      Определить главное отличие проводников  от диэлектриков.

Группа 2. Проводники в электрическом поле.

1.      Поле внутри проводника.

2.      Электростатическая индукция. Электростатическая защита. Опыт Фарадея.

Эксперимент.Взаимодействие незаряженной гильзы с  заряженной стеклянной пластины. (Гильза притянется к пластине).

Объяснение учителем или учащимися данного эксперимента.

Когда мы подносим гильзу к заряженной пластине, то под действием её электрического поля свободные электроны металлической гильзы приходят в направленное движение и собираются на левой стороне гильзы. Поэтому гильза притягивается к пластине.

Правая сторона гильзы, заряжается положительно. Внутри гильзы возникает своё электрическое поле, направленное против внешнего поля. И как только внутреннее поле станет равным внешнему полю, движение электронов прекратится. Внутри заряженного проводника поле становится равным нулю.

Опыт Фарадея.

В разделении зарядов и заключается явление электростатической индукции.Благодаря этому явлению осуществляется электростатическая защита. Если какой-либо прибор необходимо защитить от внешних электрических полей, то его помещают в проводящую оболочку.

Английский физик Майкл Фарадей провёл следующий опыт. Оклеил большую деревянную клетку листами станиоля (оловянной бумагой) и изолировал её от Земли. При помощи электрической машины Фарадей очень сильно зарядил клетку, а сам поместился в неё с чувствительным электроскопом. При этом электроскоп не показывал никакого отклонения.

Эксперимент 2

Возьмём электрометр, на стержне которого укреплена малая сфера, и поднесём к нему положительно заряженную стеклянную пластину. Под действием поля пластины стрелка электрометра отклонится от стержня. Накроем теперь сферу металлическим стаканом и так же поднесём заряженную пластину. Стрелка отклоняться не будет. Стакан  оказывает экранирующее действие. Внутри него электрического поля нет.

Группа 3. Диэлектрики в электростатическом поле.

1.Полярные и неполярные диэлектрики

2.Строение полярного диэлектрика (на примере    молекулы NaCl).Диполь.

3.Поведение полярного диэлектрика в эл поле.

 

Группа 4. Диэлектрики в электростатическом поле.

1.Поляризация диэлектрика.

2.Неполярные диэлектрики.

3.Поведение  неполярного диэлектрика в эл поле.

Эксперимент.Возьмём электрометр с металлическим диском и зарядим его положительно. Поднесём к диску лист пластика, стрелка электрометра приблизилась к стержню. Значит, диэлектрик ослабляет поле диска.

 

Диэлектрическая проницаемость среды

Знакомствоучащиеся с таблицей «Диэлектрическая проницаемость среды».

 

Выводы:

ü  Главное отличие проводников от диэлектриков — наличие свободных зарядов, которые могут перемещаться под действием электрического поля.

ü  Внутри заряженного проводника электростатическое поле отсутствует.

ü  Напряженность электростатического поля перпендикулярна поверхности проводника.

ü  Диэлектрики — это вещества, не содержащие свободных заряженных частиц.

ü  В полярных диэлектриках молекулы являются диполями, в которых центры распределения положительных и отрицательных зарядов не совпадают.

ü  Неполярные диэлектрики состоят из атомов или молекул, у которых центры распределения положительных и отрицательных зарядов совпадают.

ü  При поляризации молекулы диэлектрика ориентируются во внешнем электрическом поле.

ü  Диэлектрическая проницаемость характеризует способность диэлектрика к ослаблению внешнего поля.

Закрепление.  Проверка усвоения материала.

  1. Диэлектрик поместили в электростатическое поле, а затем разрезали на две части. Полученные половинки оказались…
  1. Какое явление называется поляризацией диэлектрика?

1.      На рисунке изображены различные вещества, внесенные в однородное электрическое поле. Стрелками показано направление линий напряженности внешнего электрического поля. Укажите диэлектрик.

  1. Какое из перечисленных веществ лишнее?
    1. Железо
    2. Резина
    3. Дерево
    4. Шёлк
  1. Напряженность электростатического поля в вакууме 20 кН/Кл. Какова напряженность этого поля в керосине, если его диэлектрическая проницаемость равна 2?

Домашнее задание🙂

В чем основное различие между проводником и диэлектриком?

проводник: Материал, содержащий подвижные электрические заряды. диэлектрик: электрически изолирующий или непроводящий материал, рассматриваемый на предмет его электрической восприимчивости (т. е. его свойства поляризации при воздействии внешнего электрического поля).

Положительные заряды внутри диэлектрика мгновенно смещаются в направлении электрического поля, а отрицательные заряды мгновенно смещаются в направлении, противоположном электрическому полю.Наличие диэлектрического материала влияет на другие электрические явления. Сила между двумя электрическими зарядами в диэлектрической среде меньше, чем в вакууме, в то время как количество энергии, хранящейся в электрическом поле на единицу объема диэлектрической среды, больше. Влияние диэлектрика на электрические явления описывается в крупном или макроскопическом масштабе с использованием таких понятий, как диэлектрическая проницаемость, диэлектрическая проницаемость и поляризация.

Диэлектрик — это проводник? Диэлектрик, изоляционный материал или очень плохой проводник электрического тока.… Когда диэлектрики помещены в электрическое поле, в них практически не протекает ток, потому что, в отличие от металлов, они не имеют слабосвязанных или свободных электронов, которые могут дрейфовать через материал.

Что такое изолятор? Знаете ответ на этот вопрос? Помогите сообществу, став участником.

Чем отличается проводник от изолятора на примере? В проводнике электрический ток может течь свободно, в изоляторе — нет.Металлы, такие как медь, являются типичными проводниками, в то время как большинство неметаллических твердых тел считаются хорошими изоляторами, имеющими чрезвычайно высокое сопротивление потоку заряда через них.

Что такое проводники и изоляторы 7? Проводник позволяет току легко проходить через него. Изоляторы не пропускают ток через него. В изоляторе отсутствуют электрические заряды.

Дополнительные вопросы

Что такое проводники и изоляторы 6 класса?

Резюме.Материалы, пропускающие электрический ток, называются проводниками. Материалы, не пропускающие электрический ток, называются изоляторами.

В чем разница между диэлектриками и изоляторами?

Материал, который накапливает электрическую энергию в электрическом поле, известен как диэлектрический материал, тогда как материал, который блокирует поток электронов, известен как изоляторы. … Диэлектрический материал накапливает электрические заряды, а изолятор блокирует электрические заряды.

Какие 10 примеров проводников?

— Серебро.
— Золото.
— Медь.
— Алюминий.
— Меркурий.
— Сталь.
— Железо.
— Морская вода.

Какие 5 примеров проводников?

— серебро.
— медь.
— золото.
— алюминий.
— железо.
— сталь.
— латунь.
— бронза.

Что такое изолятор 6 класса?

Материалы, не пропускающие электричество, называются изоляторами.Некоторые распространенные изоляторы — это стекло, воздух, пластик, хлопок, термоколь, дерево и резина. Поскольку изоляторы не проводят электричество, они используются для защиты от опасного воздействия электричества.

В чем разница между проводниками и непроводниками, приведите по два примера каждого из них?

проводников пропускают через них тепло и электричество. непроводники не пропускают тепло и электричество через них. Примеры проводников: сталь, железо и т. д.

Что такое проводник и изолятор с примером?

Примеры проводников включают металлы, водные растворы солей (т.е.е. ионные соединения, растворенные в воде), графите и человеческом теле. Примеры изоляторов включают пластмассы, пенополистирол, бумагу, резину, стекло и сухой воздух.

Почему он называется диэлектриком?

Диэлектрики — это материалы, не пропускающие ток. Их чаще называют изоляторами, потому что они являются полной противоположностью проводникам. … Это означает, что большие электрические поля создают свободные заряды (в данном случае электроны), которые могут свободно перемещаться через материал и переносить ток.

Что такое диэлектрик и его виды?

Диэлектрический материал — это материал, который плохо проводит электричество, но может поддерживать электростатические поля. … Диэлектрические материалы делятся на типы в зависимости от их состояния — твердые, жидкие или газообразные. Каждый тип имеет разные диэлектрические свойства и, в зависимости от состояния, разные области применения.

Уменьшают ли диэлектрики электрическое поле?

(b) Диэлектрик снижает напряженность электрического поля внутри конденсатора, что приводит к уменьшению напряжения между пластинами при одинаковом заряде.Конденсатор сохраняет тот же заряд при меньшем напряжении, что означает, что он имеет большую емкость из-за диэлектрика.

Есть ли у диэлектриков электрическое поле?

Диэлектрик, изоляционный материал или очень плохой проводник электрического тока. … Когда диэлектрики помещены в электрическое поле, в них практически не протекает ток, потому что, в отличие от металлов, они не имеют слабосвязанных или свободных электронов, которые могут дрейфовать через материал.

Что такое проводник 6 класса?

Материалы, через которые проходит электрический ток, называются проводниками электричества.Примеры: железный гвоздь, ключ, английская булавка, вода, человеческое тело и т. Д. … Электрические устройства состоят из проводов.

В чем принципиальная разница между проводниками и изоляторами?

В проводнике электрический ток может течь свободно, в изоляторе — нет. Металлы, такие как медь, являются типичными проводниками, в то время как большинство неметаллических твердых тел считаются хорошими изоляторами, имеющими чрезвычайно высокое сопротивление потоку заряда через них.

Как диэлектрик влияет на электрическое поле?

Как диэлектрик влияет на электрическое поле?

В чем основное отличие проводников полупроводников от изоляторов?

Уровни проводимости — основное различие между проводниками, полупроводниками и изоляторами.Проводники обладают высокой проводимостью, что означает, что они позволяют энергии, такой как электричество, тепло или звук, легко проходить через них. В то время как полупроводники допускают умеренный поток, а изоляторы обладают низкой проводимостью.

В чем разница между диэлектриком и проводником?

проводник: Материал, содержащий подвижные электрические заряды. диэлектрик: электрически изолирующий или непроводящий материал, рассматриваемый на предмет его электрической восприимчивости (т. е. его свойства поляризации при воздействии внешнего электрического поля).

Диэлектрик и проводник — в чем разница?

Dielectricnoun

(метаматериал) Электроизоляционный или непроводящий материал, рассматриваемый на предмет его электрической восприимчивости, то есть его свойства поляризации при воздействии внешнего электрического поля.

Проводник существительное

Тот, кто руководит или руководит; Руководство; директор.

Диэлектрический прилагательный

(электрически) изоляционный

Проводник существительный

(музыкальный) Лицо, которое дирижирует оркестром, хором или другим музыкальным ансамблем; профессионал, чья профессия — дирижер.

Диэлектрик существительное

Любое вещество или среда, передающие электрическую силу посредством процесса, отличного от проводимости, как в явлениях индукции; непроводник, отделяющий наэлектризованное за счет индукции тело от наэлектризованного тела.

Проводник существительное

Лицо, которое берет билеты на общественный транспорт и помогает пассажирам

‘проводник поезда; кондуктор трамвая;

Диэлектрик существительное

материал, такой как стекло или фарфор с незначительной электрической или теплопроводностью

Проводник существительное

То, что может передавать электричество, тепло, свет или звук.

Диэлектрик

В электромагнетизме диэлектрик (или диэлектрический материал) представляет собой электрический изолятор, который может поляризоваться под действием приложенного электрического поля. Когда диэлектрический материал помещается в электрическое поле, электрические заряды не протекают через материал, как в электрическом проводнике, а вместо этого лишь слегка смещаются от своих средних положений равновесия, вызывая поляризацию диэлектрика.

существительное дирижер

(математика) Идеал кольца, который измеряет, насколько далеко оно от того, чтобы быть полностью закрытым

существительное дирижер

Рифленый звук или посох, используемый для управления инструментами, такими как литонтриптические щипцы; директор.

Проводник существительное

(архитектура) Лидер.

Проводник существительное

Тот, кто или тот, кто руководит; лидер; командир; Руководство; менеджер; директор.

«Рвение, слепой проводник воли»;

Кондуктор существительное

Ответственный за общественные перевозки, такие как железнодорожный поезд или трамвай.

Дирижер существительное

Руководитель или руководитель оркестра или хора.

Проводник существительное

Вещество или тело, способное быть средой для передачи определенных сил, в частности.тепло или электричество; в частности, громоотвод.

Conductornoun

Рифленый звук или посох, используемый для управления инструментами, такими как литонтриптические щипцы и т. Д .; директор.

Conductor сущ.

То же, что и Leader.

Проводник существительное

человек, который возглавляет музыкальную группу

Проводник существительное

устройство, предназначенное для передачи электричества, тепла и т. Д.

Проводник существительное

вещество, которое легко проводит, например электричество и тепло

Проводник существительное

лицо, которое собирает плату за проезд в общественном транспорте

В чем разница между диэлектриком и проводником?

Диэлектрик — термин, связанный с проводником .

Как существительные, разница между диэлектриком

и проводником состоит в том, что диэлектрик (физика) является электрически изолирующим или непроводящим материалом, рассматриваемым на предмет его электрической восприимчивости, то есть его свойства поляризации при воздействии внешнего электрического поля, в то время как проводник — тот, кто ведет или ведет; Руководство; директор.

Как прилагательное

диэлектрик (электрически) изолирующий.

Другие сравнения: в чем разница?

Существительное

( en имя существительное )
  • (физика) Электроизоляционный или непроводящий материал, рассматриваемый на предмет его электрической восприимчивости, т.е.е. его свойство поляризации при воздействии внешнего электрического поля.
  • Синонимы
    * изолятор

    Производные условия
    * диэлектрическая постоянная * диэлектричество

    Английский

    Альтернативные формы

    * проводник ( устаревшее, )

    Существительное

    ( en имя существительное )
  • Тот, кто руководит или ведет; Руководство; директор.
  • * Драйден
  • Рвение, слепой проводник воли.
  • (музыка) Лицо, которое дирижирует оркестром, хором или другим музыкальным ансамблем; профессионал, чья профессия — дирижер.
  • Человек, покупающий билеты в общественном транспорте.
  • поезд кондуктор »; трамвай » проводник
  • То, что может передавать электричество, тепло, свет или звук.
  • (математика) Идеал кольца, который измеряет, насколько далеко оно от целого замкнутого
  • * 1988 , F van Oystaeyen, Lieven Le Bruyn, Перспективы теории колец
  • Если c — проводник , идеальный для R в R, то простые идеалы, не содержащие c, соответствуют локализациям, дающим дискретные кольца оценки.
  • Рифленый звук или посох, используемый для управления инструментами, такими как литонтриптические щипцы; директор.
  • (архитектура) Лидер.
  • Антонимы
    * непроводящий (3), непроводящий (3), изолирующий (3)

    Производные условия
    * молниеотвод

    Связанные термины
    * поведение * полупроводник, диэлектрик

    См. Также

    * контролер —-

    Быстрый ответ: в чем разница между изолятором и диэлектриком

    Материал, который хранит электрическую энергию в электрическом поле, известен как диэлектрический материал, тогда как материал, который блокирует поток электронов, известен как изоляторы.Диэлектрический материал накапливает электрические заряды, а изолятор блокирует электрические заряды.

    В чем основное различие между проводником и диэлектриком?

    проводник: Материал, содержащий подвижные электрические заряды. диэлектрик: электрически изолирующий или непроводящий материал, рассматриваемый на предмет его электрической восприимчивости (т. е. его свойства поляризации при воздействии внешнего электрического поля).

    Что означает диэлектрик?

    Диэлектрик, изолирующий материал или очень плохой проводник электрического тока.Когда диэлектрики помещены в электрическое поле, в них практически не протекает ток, потому что, в отличие от металлов, они не имеют слабосвязанных или свободных электронов, которые могут дрейфовать через материал.

    Почему он называется диэлектрическим?

    Уэвелл придумал слово «диэлектрик», объединив греческие «диа = сквозной» и «электрический». Это слово было сокращено до «диэлектрика», чтобы его было легче произносить. В отличие от электрического проводника, который исключает электрическое поле, диэлектрический материал позволяет электрическому полю проходить через него.

    Когда конденсатор подключен к батарее?

    Если незаряженный конденсатор C подключен к батарее с потенциалом V, то при зарядке пластин конденсатора протекает переходный ток. Ток от батареи прекращается, как только заряд Q на положительной пластине достигает значения Q = C × V.

    Является ли вода диэлектриком?

    Воду можно рассматривать как диэлектрик, поскольку она имеет хорошее значение относительной диэлектрической проницаемости (около 80 градусов при 20 ° C). Вода оказывается диэлектриком из-за связанной с ней диэлектрической поляризации (это электрический диполь, очень полярная молекула и даже вращается, выравниваясь в направлении поля).

    Что подразумевается под диэлектрическими потерями?

    Диэлектрические потери, потеря энергии, которая идет на нагрев диэлектрического материала в переменном электрическом поле. Например, конденсатор, включенный в цепь переменного тока, попеременно заряжается и разряжается каждый полупериод. Диэлектрические потери зависят от частоты и материала диэлектрика.

    Каковы диэлектрические свойства материалов?

    Диэлектрические свойства материалов определяются как молекулярные свойства, которые являются фундаментальными для всех материалов, способных вызывать движение электронов, приводящее к поляризации внутри материала при воздействии внешнего электрического поля.

    Что такое диэлектрик?

    Аммиак и вода — хороший пример полярных диэлектриков. Неполярные диэлектрики: неполярные диэлектрики — это материалы, которые не имеют собственного дипольного момента. Таким образом, положительный и отрицательный заряды не разделены небольшим расстоянием в отсутствие электрического поля.

    Почему не все изоляторы являются диэлектриками, а все диэлектрические материалы являются изоляторами?

    Хотя все диэлектрики являются изоляторами (они не допускают протекания через них электрических зарядов), все изоляторы не являются диэлектриками, потому что они не могут накапливать заряды, в отличие от диэлектриков.

    Должен ли настоящий диэлектрик быть идеальным изолятором?

    Материал, который накапливает электрическую энергию в электрическом поле, известен как диэлектрический материал, тогда как материал, блокирующий поток электронов, известен как изоляторы. Диэлектрический материал имеет высокую диэлектрическую проницаемость, тогда как изолятор имеет низкую диэлектрическую проницаемость.

    Является ли медь диэлектрическим материалом?

    Медь не является диэлектриком.

    Что такое диэлектрик. Объясните на примере?

    Диэлектрики — непроводящие вещества.Они являются изоляционными материалами и плохо проводят электрический ток. Диэлектрические материалы могут удерживать электростатический заряд, рассеивая при этом минимальную энергию в виде тепла. Примерами диэлектрика являются слюда, пластмассы, стекло, фарфор и различные оксиды металлов.

    Увеличивает ли диэлектрик разность потенциалов?

    Когда кусок изолятора вставляется в конденсатор, мы называем изолятор диэлектриком. Чистый эффект диэлектрика заключается в увеличении количества заряда, который конденсатор может хранить при заданной разности потенциалов.

    Почему диэлектрик увеличивает емкость?

    (a) Молекулы изоляционного материала между пластинами конденсатора поляризованы заряженными пластинами. Это создает слой противоположного заряда на поверхности диэлектрика, который притягивает больше заряда к пластине, увеличивая ее емкость.

    Может ли диэлектрик проводить электричество?

    Диэлектрик — это электрический изолятор, который можно поляризовать для проведения электричества, поместив в электрическое поле.Диэлектрические материалы имеют слабосвязанные электроны, которые дрейфуют от материала среднего положения равновесия при приложении электрического тока или при помещении в электрическое поле.

    Является ли древесина диэлектрическим материалом?

    Дерево — это материал со сложными несимметричными молекулами и неоднородной структурой, поэтому можно ожидать, что он будет иметь большую диэлектрическую проницаемость.

    В чем разница между конденсатором и диэлектриком?

    Конденсатор — это электрическое устройство, которое накапливает электрический заряд, а диэлектрик — это материал, не пропускающий ток.Диэлектрики часто называют изоляторами, поскольку они противоположны проводникам.

    Что делает диэлектрик хорошим?

    Все диэлектрические материалы являются изоляторами, но хороший диэлектрик — это тот, который легко поляризуется. Степень поляризации, возникающая при приложении определенного напряжения к объекту, влияет на количество электрической энергии, которая хранится в электрическом поле.

    Почему важна диэлектрическая проницаемость?

    Диэлектрический материал используется для разделения проводящих пластин конденсатора.Этот изоляционный материал в значительной степени определяет свойства компонента. Диэлектрическая постоянная материала определяет количество энергии, которое конденсатор может хранить при приложении напряжения.

    Конденсаторы облака?

    Облака и земля могут действовать в унисон, имитируя огромный естественный конденсатор. Воздух между облаками и землей становится диэлектриком этого природного конденсатора. Электростатическое поле между облаками и землей может производить ионы и свободные электроны в воздухе.

    Почему изоляторы называют диэлектриками?

    Диэлектрики — это материалы, не пропускающие ток. Их чаще называют изоляторами, потому что они являются полной противоположностью проводникам. Это означает, что большие электрические поля создают свободные заряды (в данном случае электроны), которые могут свободно перемещаться через материал и переносить ток.

    Какие два типа диэлектрика?

    Есть два типа диэлектриков — неполярный диэлектрик и полярный диэлектрик.

    Все ли изоляторы диэлектрические?

    Все диэлектрики будут изоляторами, но все изоляторы не будут диэлектриками. Итак, в чем разница между диэлектриком и изолятором? Изоляторы — это материалы, которые не проводят электричество в электрическом поле, так как в них нет свободных электронов.

    Какова функция диэлектрика?

    Диэлектрики в конденсаторах служат трем целям: предотвращать соприкосновение проводящих пластин, позволяя уменьшить расстояние между пластинами и, следовательно, увеличить емкость; увеличить эффективную емкость за счет уменьшения напряженности электрического поля, что означает получение такого же заряда при более низком напряжении; а также.

    Увеличивает ли диэлектрик запасенную энергию?

    Введение диэлектрика увеличивает емкость, уменьшая запасенную в конденсаторе энергию. Конденсатор действительно работает, протягивая диэлектрик между пластинами, уменьшая запасенную энергию.

    Разница между диэлектриком и изолятором (с таблицей)

    Диэлектрик и изоляторы — это два изолятора, но с совершенно разными функциями и работой. Диэлектрический изолятор позволяет и даже экономит электричество, в то время как изоляторы препятствуют прохождению электричества через него и даже противостоят теплу.Хотя, будучи изолятором, оба они работают противоположно и имеют совершенно разное применение. Один используется для экономии электричества, а другой используется для сопротивления электричеству и теплу.

    Диэлектрик против изолятора

    Разница между диэлектриком и изолятором заключается в том, что материал, который накапливает или сохраняет электрическую энергию в электрическом поле, является диэлектрическим материалом, а с другой стороны, материалом, который блокирует поток электронов. в электрическом поле — изолятор.

    Диэлектрический материал может поляризоваться в присутствии электрического поля, в то время как изоляторы, с другой стороны, не поляризованы. Говоря о диэлектрической проницаемости, диэлектрические имеют их большое количество, а изоляторы имеют сравнительно низкую диэлектрическую проницаемость.

    Электрические заряды накапливаются в диэлектрических материалах, а в изоляторах они блокируются. Изоляторы обычно используются в проводах и кабелях, поскольку они предотвращают попадание электричества, поэтому нет шансов получить удар электрическим током, пока в конденсаторе обычно используется диэлектрический материал.

    Диэлектрические материалы используются для проверки того, достаточно ли изоляция компонента защищает пользователей от поражения электрическим током. В то время как изоляторы в основном используются перед любыми испытаниями высокого напряжения, чтобы исключить любые загрязнения в изоляции электричества. Диэлектрики способны выдерживать высокие электрические нагрузки без какой-либо проводимости. Но изоляторы ограничивают любой перенос или поток электронов.

    Диэлектрики — это просто изоляторы, в которых нет свободных электронов.При приложении к ним электрического поля диэлектрики легко поляризуются. В то время как изолятор — это материал, который позволяет передавать тепло или электричество от него. Некоторые из изоляционных материалов включают бумагу, стекло, масло, резину и пластик. Хотя вакуум также является изолятором, его нельзя рассматривать как материал

    Таблица сравнения диэлектрика и изолятора

    Параметры сравнения Диэлектрик Определители изоляторов
    Это электрический изолятор, способный выдерживать высокое электрическое напряжение без какой-либо проводимости. Это материалы или устройства, ограничивающие передачу тепла или электричества .
    Использование Он используется для проверки того, достаточно ли изоляция компонента защищает пользователей от поражения электрическим током. Он в основном используется перед любыми испытаниями с высоким потенциалом, чтобы исключить любые загрязнения в электрической изоляции .
    Поляризация В присутствии электрического поля диэлектрики могут быть очень легко поляризованы. Изоляторы нельзя поляризовать.
    Число диэлектрических постоянных Диэлектрики имеют большое количество диэлектрических постоянных. Изоляторы имеют сравнительно низкую диэлектрическую проницаемость.
    Пример Слюда, пластик и оксиды различных материалов. Резина, стекло, алмаз, дерево и масло

    Что такое диэлектрик?

    Диэлектрик — это в основном материал с низкой электропроводностью, но он наследует способность сохранять электрический заряд.По сути, это просто изоляторы, в которых нет свободных электронов. При приложении к ним электрического поля диэлектрики легко поляризуются. Таким образом, можно сказать, что их поведение в области электричества полностью отличается от поведения проводников.

    Есть два типа диэлектрических материалов — полярные и неполярные. Полярные постоянны в электрическом диполе, и их поляризация зависит от температуры. В то время как неполярные, индуцированный электрический диполь и их поляризация полностью не зависят от температуры.

    Что такое изолятор?

    Изолятор — это материал, который позволяет отводить от него тепло или электричество. Некоторые из изоляционных материалов включают бумагу, стекло, масло, резину и пластик. Хотя вакуум также является изолятором, его нельзя рассматривать как материал. В большинстве случаев все электрические материалы покрыты изоляцией, чтобы избежать попадания в них электрического тока.

    Обычно изоляторы рассчитаны на несколько сотен вольт, но некоторые из них, которые используются для распределения энергии, даже рассчитаны на сотни тысяч вольт.Чтобы сделать любой непреднамеренный контакт, изоляторы поддерживаются или удерживаются от электрических проводников.

    Основное различие между диэлектриком и изолятором
    1. Основное и существенное различие между диэлектриком и изолятором состоит в том, что диэлектрик позволяет и сохраняет поток электричества в нем и через него, в то время как изолятор, с другой стороны, блокирует и предотвращает отток электронов и электричества от него.
    2. Диэлектрический материал может поляризоваться в присутствии электрического поля, в то время как изоляторы, с другой стороны, не поляризованы.
    3. Молекулы в диэлектрическом материале связаны очень неделями, тогда как молекулы в изоляторе прочно связаны друг с другом.
    4. Диэлектрик имеет высокую диэлектрическую проницаемость, тогда как изоляторы имеют сравнительно низкую диэлектрическую проницаемость.
    5. Изоляторы обычно используются в проводах и кабелях, поскольку они предотвращают возникновение электричества, так что нет шансов получить удар электрическим током, пока в конденсаторе обычно используется диэлектрический материал.
    6. Примеры изоляторов, препятствующих прохождению и передаче электричества: воздух, стекло, пластик, сухое дерево и медь.Пример диэлектрика — конденсатор.

    Заключение

    Таким образом, мы пришли к выводу, что диэлектрик — это не что иное, как изолирующий материал, но тот, который хранит и пропускает электричество, в то время как изоляторы, с другой стороны, являются материалами с нулевым уровнем электропроводности. в них и не допускает протекания или передачи электричества через них, фактически, они создают препятствия для потока и передачи электрического тока.

    Диэлектрические материалы используются для проверки того, достаточно ли изоляция компонента защищает пользователей от поражения электрическим током. В то время как изоляторы в основном используются перед любыми испытаниями с высоким потенциалом, чтобы исключить любые загрязнения в изоляции электричества. Диэлектрики способны выдерживать высокое электрическое напряжение без какой-либо проводимости. Но изоляторы ограничивают любой перенос или поток электронов.

    Ссылки
    1. https: // www.sciencedirect.com/science/article/pii/S0924013607004657
    2. https://link.springer.com/chapter/10.1007/978-3-642-77453-9_8

    Разница между диэлектриком и изолятором по сравнительной таблице

    Диэлектрик и изолятор различаются по сферам применения. Одно из основных различий между диэлектриком и изолятором заключается в том, что диэлектрик накапливает электрические заряды, в то время как изолятор препятствует потоку электронов. Некоторые другие различия между ними объясняются ниже в сравнительной таблице.

    Содержание: диэлектрик против изолятора

    1. Сравнительная таблица
    2. Определение
    3. Ключевые отличия

    Сравнительная таблица

    Основа для сравнения Диэлектрик Изолятор
    Определение Материал, в котором электрическое поле может развиваться с минимальными потерями энергии, известен как диэлектрик. Вещество с низкой проводимостью, препятствующее прохождению тока, известное как изолятор.
    Поляризация Поляризация в электрическом поле. Не может поляризоваться
    Связь Слабая связь по сравнению с изолятором. Ковалентная связь
    Диэлектрическая проницаемость Высокая Низкая
    Заряды Хранить заряды Препятствие для зарядов.
    Пример Сухой воздух, вакуум, дистиллированная вода и т. Д. Хлопок, пластик, слюда и т. Д.
    Применение Конденсатор, силовой кабель и т. Д. Проводящие провода в высоковольтных системах и т. Д.

    Определение диэлектрика

    Диэлектрический материал представляет собой тип изолятора, который имеет несколько свободных электронов. Он становится поляризованным в присутствии электрического поля. Поляризация — это свойство материала, в котором положительные и отрицательные заряды материала смещены в противоположном направлении.Поляризация уменьшает общее поле материала.

    Накопление и рассеяние электрической энергии — основные свойства диэлектрического материала. Проводимость идеального диэлектрического материала равна нулю. Типичным примером диэлектрика является конденсатор. Поляризация между параллельными пластинами конденсатора увеличивает площадь поверхности емкости.

    Определение изолятора

    Материал, не пропускающий электрический ток, известен как изолятор.Изоляционный материал не имеет свободных электронов, потому что их молекулы имеют прочную ковалентную связь. Удельное сопротивление материала очень высокое по сравнению с другим материалом. Удельное сопротивление — это свойство материала, которое показывает сильное препятствие потоку зарядов.

    Эбонит, бумага, дерево, пластик — вот некоторые примеры изоляторов.

    Ключевые различия между диэлектриком и изолятором

    1. Материал, который накапливает электрическую энергию в электрическом поле, известен как диэлектрический материал, тогда как материал, который блокирует поток электронов, известен как изоляторы.
    2. Диэлектрический материал поляризуется в присутствии электрического поля, тогда как изоляторы не поляризованы.
      Примечание: Поляризация — это свойство материала, в котором положительный и отрицательный заряды движутся в противоположных направлениях.
    3. Молекулы диэлектрического материала связаны слабо, тогда как молекулы изоляторов прочно связаны друг с другом по сравнению с диэлектриком.
    4. Диэлектрический материал имеет высокую диэлектрическую проницаемость, тогда как изолятор имеет низкую диэлектрическую проницаемость.Диэлектрическая проницаемость измеряет запасающую способность материала.
    5. Диэлектрический материал накапливает электрические заряды, а изолятор блокирует электрические заряды.
    6. Сухой воздух, вакуум и дистиллированная вода являются примерами диэлектрика, тогда как хлопок, пластик, резина являются примерами изоляторов.
    7. Наиболее распространенным применением диэлектрика является конденсатор, а изоляторы используются в проводниках и кабелях.

    Почти весь изолятор ведет себя как диэлектрик, но не все диэлектрики как изоляторы.

    Разница между изолятором и диэлектриком

    Ключевое отличие: Изолятор — это материал, не проводящий электричество. С другой стороны, диэлектрики — это изоляторы, поляризуемые при приложении электрического поля. Изоляторы — полная противоположность проводникам.

    Изоляторы относятся к тем материалам, в которых электрический ток не может свободно течь. В отличие от проводников изоляторы обеспечивают большее сопротивление прохождению электрического тока.Электроны в изоляторах тесно и прочно связаны с атомами ионными или ковалентными связями, и это основная причина того, что через изоляторы не течет ток. Стекло и резина являются примерами изоляторов.

    Диэлектрики также являются типами изоляторов, но они обладают способностью передавать заряд при приложении внешнего электрического поля. Это происходит из-за производства индуцированных зарядов, возникающих из-за приложения электрического поля. Эти наведенные заряды отображают проводящие свойства.

    Проще говоря, диэлектрики — это те изоляторы, которые можно поляризовать. В этих материалах электроны связаны с ядром и поэтому обладают очень малым движением. При приложении внешнего напряжения ядро ​​и атомы притягиваются к отрицательной и положительной сторонам соответственно. Фарфор (керамика), слюда, стекло и пластмассы — все это примеры диэлектрических материалов.

    Когда диэлектрик помещается в заряженный конденсатор, он уменьшает разность потенциалов между двумя пластинами.Мера способности материала генерировать электрическое поле известна как диэлектрическая постоянная или относительная диэлектрическая проницаемость, которая обозначается греческой буквой эпсилон.

    Следовательно, все диэлектрики являются изоляторами, но не все изоляторы диэлектриками.

    Сравнение изолятора и диэлектрика:

    Изолятор

    Диэлектрик

    Определения

    Материалы, препятствующие протеканию в них электрического тока

    Материалы, которые являются изоляторами, но поляризуются под действием внешнего электрического поля

    Диэлектрическая проницаемость

    Сравнительно низкий

    Сравнительно высокий

    Примеры

    Стекло, фарфор, пластик, резина

    воздух, слюда, керамика, бумага, полиэстер

    Важная характеристика

    В изоляторах проводимость σ << 1

    Напротив кондуктора

    Электрическая прочность различается для разных диэлектриков

    Типы

    • Тип штифта — крепится к траверсе на опоре
    • Подвесной тип — дешевле изоляторов штыревого типа и состоит из нескольких фарфоровых дисков
    • Изолятор деформации — он был создан для работы в условиях механического напряжения
    • Скоба изоляторы — используются для низковольтных распределительных линий
    • Неполярные — центры положительных и отрицательных зарядов в молекуле совпадают, и электрический диполь не образуется
    • Полярные диэлектрики — в указанном выше состоянии, если сформирован электрический диполь — например, HCL
    .

    Добавить комментарий

    Ваш адрес email не будет опубликован. Обязательные поля помечены *